Stay ahead of learning milestones! Enroll in a class over the summer!

G
Topic
First Poster
Last Poster
k a May Highlights and 2025 AoPS Online Class Information
jlacosta   0
May 1, 2025
May is an exciting month! National MATHCOUNTS is the second week of May in Washington D.C. and our Founder, Richard Rusczyk will be presenting a seminar, Preparing Strong Math Students for College and Careers, on May 11th.

Are you interested in working towards MATHCOUNTS and don’t know where to start? We have you covered! If you have taken Prealgebra, then you are ready for MATHCOUNTS/AMC 8 Basics. Already aiming for State or National MATHCOUNTS and harder AMC 8 problems? Then our MATHCOUNTS/AMC 8 Advanced course is for you.

Summer camps are starting next month at the Virtual Campus in math and language arts that are 2 - to 4 - weeks in duration. Spaces are still available - don’t miss your chance to have an enriching summer experience. There are middle and high school competition math camps as well as Math Beasts camps that review key topics coupled with fun explorations covering areas such as graph theory (Math Beasts Camp 6), cryptography (Math Beasts Camp 7-8), and topology (Math Beasts Camp 8-9)!

Be sure to mark your calendars for the following upcoming events:
[list][*]May 9th, 4:30pm PT/7:30pm ET, Casework 2: Overwhelming Evidence — A Text Adventure, a game where participants will work together to navigate the map, solve puzzles, and win! All are welcome.
[*]May 19th, 4:30pm PT/7:30pm ET, What's Next After Beast Academy?, designed for students finishing Beast Academy and ready for Prealgebra 1.
[*]May 20th, 4:00pm PT/7:00pm ET, Mathcamp 2025 Qualifying Quiz Part 1 Math Jam, Problems 1 to 4, join the Canada/USA Mathcamp staff for this exciting Math Jam, where they discuss solutions to Problems 1 to 4 of the 2025 Mathcamp Qualifying Quiz!
[*]May 21st, 4:00pm PT/7:00pm ET, Mathcamp 2025 Qualifying Quiz Part 2 Math Jam, Problems 5 and 6, Canada/USA Mathcamp staff will discuss solutions to Problems 5 and 6 of the 2025 Mathcamp Qualifying Quiz![/list]
Our full course list for upcoming classes is below:
All classes run 7:30pm-8:45pm ET/4:30pm - 5:45pm PT unless otherwise noted.

Introductory: Grades 5-10

Prealgebra 1 Self-Paced

Prealgebra 1
Tuesday, May 13 - Aug 26
Thursday, May 29 - Sep 11
Sunday, Jun 15 - Oct 12
Monday, Jun 30 - Oct 20
Wednesday, Jul 16 - Oct 29

Prealgebra 2 Self-Paced

Prealgebra 2
Wednesday, May 7 - Aug 20
Monday, Jun 2 - Sep 22
Sunday, Jun 29 - Oct 26
Friday, Jul 25 - Nov 21

Introduction to Algebra A Self-Paced

Introduction to Algebra A
Sunday, May 11 - Sep 14 (1:00 - 2:30 pm ET/10:00 - 11:30 am PT)
Wednesday, May 14 - Aug 27
Friday, May 30 - Sep 26
Monday, Jun 2 - Sep 22
Sunday, Jun 15 - Oct 12
Thursday, Jun 26 - Oct 9
Tuesday, Jul 15 - Oct 28

Introduction to Counting & Probability Self-Paced

Introduction to Counting & Probability
Thursday, May 15 - Jul 31
Sunday, Jun 1 - Aug 24
Thursday, Jun 12 - Aug 28
Wednesday, Jul 9 - Sep 24
Sunday, Jul 27 - Oct 19

Introduction to Number Theory
Friday, May 9 - Aug 1
Wednesday, May 21 - Aug 6
Monday, Jun 9 - Aug 25
Sunday, Jun 15 - Sep 14
Tuesday, Jul 15 - Sep 30

Introduction to Algebra B Self-Paced

Introduction to Algebra B
Tuesday, May 6 - Aug 19
Wednesday, Jun 4 - Sep 17
Sunday, Jun 22 - Oct 19
Friday, Jul 18 - Nov 14

Introduction to Geometry
Sunday, May 11 - Nov 9
Tuesday, May 20 - Oct 28
Monday, Jun 16 - Dec 8
Friday, Jun 20 - Jan 9
Sunday, Jun 29 - Jan 11
Monday, Jul 14 - Jan 19

Paradoxes and Infinity
Mon, Tue, Wed, & Thurs, Jul 14 - Jul 16 (meets every day of the week!)

Intermediate: Grades 8-12

Intermediate Algebra
Sunday, Jun 1 - Nov 23
Tuesday, Jun 10 - Nov 18
Wednesday, Jun 25 - Dec 10
Sunday, Jul 13 - Jan 18
Thursday, Jul 24 - Jan 22

Intermediate Counting & Probability
Wednesday, May 21 - Sep 17
Sunday, Jun 22 - Nov 2

Intermediate Number Theory
Sunday, Jun 1 - Aug 24
Wednesday, Jun 18 - Sep 3

Precalculus
Friday, May 16 - Oct 24
Sunday, Jun 1 - Nov 9
Monday, Jun 30 - Dec 8

Advanced: Grades 9-12

Olympiad Geometry
Tuesday, Jun 10 - Aug 26

Calculus
Tuesday, May 27 - Nov 11
Wednesday, Jun 25 - Dec 17

Group Theory
Thursday, Jun 12 - Sep 11

Contest Preparation: Grades 6-12

MATHCOUNTS/AMC 8 Basics
Friday, May 23 - Aug 15
Monday, Jun 2 - Aug 18
Thursday, Jun 12 - Aug 28
Sunday, Jun 22 - Sep 21
Tues & Thurs, Jul 8 - Aug 14 (meets twice a week!)

MATHCOUNTS/AMC 8 Advanced
Sunday, May 11 - Aug 10
Tuesday, May 27 - Aug 12
Wednesday, Jun 11 - Aug 27
Sunday, Jun 22 - Sep 21
Tues & Thurs, Jul 8 - Aug 14 (meets twice a week!)

AMC 10 Problem Series
Friday, May 9 - Aug 1
Sunday, Jun 1 - Aug 24
Thursday, Jun 12 - Aug 28
Tuesday, Jun 17 - Sep 2
Sunday, Jun 22 - Sep 21 (1:00 - 2:30 pm ET/10:00 - 11:30 am PT)
Monday, Jun 23 - Sep 15
Tues & Thurs, Jul 8 - Aug 14 (meets twice a week!)

AMC 10 Final Fives
Sunday, May 11 - Jun 8
Tuesday, May 27 - Jun 17
Monday, Jun 30 - Jul 21

AMC 12 Problem Series
Tuesday, May 27 - Aug 12
Thursday, Jun 12 - Aug 28
Sunday, Jun 22 - Sep 21
Wednesday, Aug 6 - Oct 22

AMC 12 Final Fives
Sunday, May 18 - Jun 15

AIME Problem Series A
Thursday, May 22 - Jul 31

AIME Problem Series B
Sunday, Jun 22 - Sep 21

F=ma Problem Series
Wednesday, Jun 11 - Aug 27

WOOT Programs
Visit the pages linked for full schedule details for each of these programs!


MathWOOT Level 1
MathWOOT Level 2
ChemWOOT
CodeWOOT
PhysicsWOOT

Programming

Introduction to Programming with Python
Thursday, May 22 - Aug 7
Sunday, Jun 15 - Sep 14 (1:00 - 2:30 pm ET/10:00 - 11:30 am PT)
Tuesday, Jun 17 - Sep 2
Monday, Jun 30 - Sep 22

Intermediate Programming with Python
Sunday, Jun 1 - Aug 24
Monday, Jun 30 - Sep 22

USACO Bronze Problem Series
Tuesday, May 13 - Jul 29
Sunday, Jun 22 - Sep 1

Physics

Introduction to Physics
Wednesday, May 21 - Aug 6
Sunday, Jun 15 - Sep 14
Monday, Jun 23 - Sep 15

Physics 1: Mechanics
Thursday, May 22 - Oct 30
Monday, Jun 23 - Dec 15

Relativity
Mon, Tue, Wed & Thurs, Jun 23 - Jun 26 (meets every day of the week!)
0 replies
jlacosta
May 1, 2025
0 replies
k i Adding contests to the Contest Collections
dcouchman   1
N Apr 5, 2023 by v_Enhance
Want to help AoPS remain a valuable Olympiad resource? Help us add contests to AoPS's Contest Collections.

Find instructions and a list of contests to add here: https://artofproblemsolving.com/community/c40244h1064480_contests_to_add
1 reply
dcouchman
Sep 9, 2019
v_Enhance
Apr 5, 2023
k i Zero tolerance
ZetaX   49
N May 4, 2019 by NoDealsHere
Source: Use your common sense! (enough is enough)
Some users don't want to learn, some other simply ignore advises.
But please follow the following guideline:


To make it short: ALWAYS USE YOUR COMMON SENSE IF POSTING!
If you don't have common sense, don't post.


More specifically:

For new threads:


a) Good, meaningful title:
The title has to say what the problem is about in best way possible.
If that title occured already, it's definitely bad. And contest names aren't good either.
That's in fact a requirement for being able to search old problems.

Examples:
Bad titles:
- "Hard"/"Medium"/"Easy" (if you find it so cool how hard/easy it is, tell it in the post and use a title that tells us the problem)
- "Number Theory" (hey guy, guess why this forum's named that way¿ and is it the only such problem on earth¿)
- "Fibonacci" (there are millions of Fibonacci problems out there, all posted and named the same...)
- "Chinese TST 2003" (does this say anything about the problem¿)
Good titles:
- "On divisors of a³+2b³+4c³-6abc"
- "Number of solutions to x²+y²=6z²"
- "Fibonacci numbers are never squares"


b) Use search function:
Before posting a "new" problem spend at least two, better five, minutes to look if this problem was posted before. If it was, don't repost it. If you have anything important to say on topic, post it in one of the older threads.
If the thread is locked cause of this, use search function.

Update (by Amir Hossein). The best way to search for two keywords in AoPS is to input
[code]+"first keyword" +"second keyword"[/code]
so that any post containing both strings "first word" and "second form".


c) Good problem statement:
Some recent really bad post was:
[quote]$lim_{n\to 1}^{+\infty}\frac{1}{n}-lnn$[/quote]
It contains no question and no answer.
If you do this, too, you are on the best way to get your thread deleted. Write everything clearly, define where your variables come from (and define the "natural" numbers if used). Additionally read your post at least twice before submitting. After you sent it, read it again and use the Edit-Button if necessary to correct errors.


For answers to already existing threads:


d) Of any interest and with content:
Don't post things that are more trivial than completely obvious. For example, if the question is to solve $x^{3}+y^{3}=z^{3}$, do not answer with "$x=y=z=0$ is a solution" only. Either you post any kind of proof or at least something unexpected (like "$x=1337, y=481, z=42$ is the smallest solution). Someone that does not see that $x=y=z=0$ is a solution of the above without your post is completely wrong here, this is an IMO-level forum.
Similar, posting "I have solved this problem" but not posting anything else is not welcome; it even looks that you just want to show off what a genius you are.

e) Well written and checked answers:
Like c) for new threads, check your solutions at least twice for mistakes. And after sending, read it again and use the Edit-Button if necessary to correct errors.



To repeat it: ALWAYS USE YOUR COMMON SENSE IF POSTING!


Everything definitely out of range of common sense will be locked or deleted (exept for new users having less than about 42 posts, they are newbies and need/get some time to learn).

The above rules will be applied from next monday (5. march of 2007).
Feel free to discuss on this here.
49 replies
ZetaX
Feb 27, 2007
NoDealsHere
May 4, 2019
Cyclic equality implies equal sum of squares
blackbluecar   33
N 23 minutes ago by Vivaandax
Source: 2021 Iberoamerican Mathematical Olympiad, P4
Let $a,b,c,x,y,z$ be real numbers such that

\[ a^2+x^2=b^2+y^2=c^2+z^2=(a+b)^2+(x+y)^2=(b+c)^2+(y+z)^2=(c+a)^2+(z+x)^2 \]
Show that $a^2+b^2+c^2=x^2+y^2+z^2$.
33 replies
blackbluecar
Oct 21, 2021
Vivaandax
23 minutes ago
3-var inequality
sqing   3
N 36 minutes ago by ytChen
Source: Own
Let $ a,b>0 $ and $\frac{1}{a^2+3}+ \frac{1}{b^2+ 3} \leq \frac{1}{2} . $ Prove that
$$a^2+ab+b^2\geq 3$$$$a^2-ab+b^2 \geq 1 $$Let $ a,b>0 $ and $\frac{1}{a^3+3}+ \frac{1}{b^3+ 3}\leq \frac{1}{2} . $ Prove that
$$a^3+ab+b^3 \geq 3$$$$ a^3-ab+b^3\geq 1 $$
3 replies
sqing
May 7, 2025
ytChen
36 minutes ago
An I for an I
Eyed   67
N an hour ago by AR17296174
Source: 2020 ISL G8
Let $ABC$ be a triangle with incenter $I$ and circumcircle $\Gamma$. Circles $\omega_{B}$ passing through $B$ and $\omega_{C}$ passing through $C$ are tangent at $I$. Let $\omega_{B}$ meet minor arc $AB$ of $\Gamma$ at $P$ and $AB$ at $M\neq B$, and let $\omega_{C}$ meet minor arc $AC$ of $\Gamma$ at $Q$ and $AC$ at $N\neq C$. Rays $PM$ and $QN$ meet at $X$. Let $Y$ be a point such that $YB$ is tangent to $\omega_{B}$ and $YC$ is tangent to $\omega_{C}$.

Show that $A,X,Y$ are collinear.
67 replies
Eyed
Jul 20, 2021
AR17296174
an hour ago
IMO 2018 Problem 1
juckter   169
N 2 hours ago by Thelink_20
Let $\Gamma$ be the circumcircle of acute triangle $ABC$. Points $D$ and $E$ are on segments $AB$ and $AC$ respectively such that $AD = AE$. The perpendicular bisectors of $BD$ and $CE$ intersect minor arcs $AB$ and $AC$ of $\Gamma$ at points $F$ and $G$ respectively. Prove that lines $DE$ and $FG$ are either parallel or they are the same line.

Proposed by Silouanos Brazitikos, Evangelos Psychas and Michael Sarantis, Greece
169 replies
juckter
Jul 9, 2018
Thelink_20
2 hours ago
Urgent. Need them quick
sealight2107   2
N 2 hours ago by Bergo1305
With $a,b,c>1$ and $a+b+c=2abc$. Prove that:
$\sqrt[3]{ab-1}+\sqrt[3]{bc-1}+\sqrt[3]{ca-1} \le \sqrt[3]{(a+b+c)^2}$
2 replies
sealight2107
Yesterday at 4:58 PM
Bergo1305
2 hours ago
Croatian mathematical olympiad, day 1, problem 2
Matematika   6
N 2 hours ago by Cqy00000000
There were finitely many persons at a party among whom some were friends. Among any $4$ of them there were either $3$ who were all friends among each other or $3$ who weren't friend with each other. Prove that you can separate all the people at the party in two groups in such a way that in the first group everyone is friends with each other and that all the people in the second group are not friends to anyone else in second group. (Friendship is a mutual relation).
6 replies
Matematika
Apr 10, 2011
Cqy00000000
2 hours ago
Game
Pascual2005   27
N 3 hours ago by HamstPan38825
Source: Colombia TST, IMO ShortList 2004, combinatorics problem 5
$A$ and $B$ play a game, given an integer $N$, $A$ writes down $1$ first, then every player sees the last number written and if it is $n$ then in his turn he writes $n+1$ or $2n$, but his number cannot be bigger than $N$. The player who writes $N$ wins. For which values of $N$ does $B$ win?

Proposed by A. Slinko & S. Marshall, New Zealand
27 replies
Pascual2005
Jun 7, 2005
HamstPan38825
3 hours ago
Lines concur on bisector of BAC
Invertibility   2
N 5 hours ago by NO_SQUARES
Source: Slovenia 2025 TST 3 P2
Let $\Omega$ be the circumcircle of a scalene triangle $ABC$. Let $\omega$ be a circle internally tangent to $\Omega$ in $A$. Tangents from $B$ touch $\omega$ in $P$ and $Q$, such that $P$ lies in the interior of $\triangle{}ABC$. Similarly, tangents from $C$ touch $\omega$ in $R$ and $S$, such that $R$ lies in the interior of $\triangle{}ABC$.

Prove that $PS$ and $QR$ concur on the bisector of $\angle{}BAC$.
2 replies
Invertibility
5 hours ago
NO_SQUARES
5 hours ago
Why is the old one deleted?
EeEeRUT   16
N 5 hours ago by ravengsd
Source: EGMO 2025 P1
For a positive integer $N$, let $c_1 < c_2 < \cdots < c_m$ be all positive integers smaller than $N$ that are coprime to $N$. Find all $N \geqslant 3$ such that $$\gcd( N, c_i + c_{i+1}) \neq 1$$for all $1 \leqslant i \leqslant m-1$

Here $\gcd(a, b)$ is the largest positive integer that divides both $a$ and $b$. Integers $a$ and $b$ are coprime if $\gcd(a, b) = 1$.

Proposed by Paulius Aleknavičius, Lithuania
16 replies
EeEeRUT
Apr 16, 2025
ravengsd
5 hours ago
angle chasing with 2 midpoints, equal angles given and wanted
parmenides51   5
N 6 hours ago by breloje17fr
Source: Ukrainian Geometry Olympiad 2017, IX p1, X p1, XI p1
In the triangle $ABC$, ${{A}_{1}}$ and ${{C}_{1}} $ are the midpoints of sides $BC $ and $AB$ respectively. Point $P$ lies inside the triangle. Let $\angle BP {{C}_{1}} = \angle PCA$. Prove that $\angle BP {{A}_{1}} = \angle PAC $.
5 replies
parmenides51
Dec 11, 2018
breloje17fr
6 hours ago
Problem 4 of Finals
GeorgeRP   2
N Yesterday at 7:00 PM by Assassino9931
Source: XIII International Festival of Young Mathematicians Sozopol 2024, Theme for 10-12 grade
The diagonals \( AD \), \( BE \), and \( CF \) of a hexagon \( ABCDEF \) inscribed in a circle \( k \) intersect at a point \( P \), and the acute angle between any two of them is \( 60^\circ \). Let \( r_{AB} \) be the radius of the circle tangent to segments \( PA \) and \( PB \) and internally tangent to \( k \); the radii \( r_{BC} \), \( r_{CD} \), \( r_{DE} \), \( r_{EF} \), and \( r_{FA} \) are defined similarly. Prove that
\[
r_{AB}r_{CD} + r_{CD}r_{EF} + r_{EF}r_{AB} = r_{BC}r_{DE} + r_{DE}r_{FA} + r_{FA}r_{BC}.
\]
2 replies
GeorgeRP
Sep 10, 2024
Assassino9931
Yesterday at 7:00 PM
Interesting functional equation with geometry
User21837561   3
N Yesterday at 6:34 PM by Double07
Source: BMOSL 2024 G7
For an acute triangle $ABC$, let $O$ be the circumcentre, $H$ be the orthocentre, and $G$ be the centroid.
Let $f:\pi\rightarrow\mathbb R$ satisfy the following condition:
$f(A)+f(B)+f(C)=f(O)+f(G)+f(H)$
Prove that $f$ is constant.
3 replies
User21837561
Yesterday at 8:14 AM
Double07
Yesterday at 6:34 PM
greatest volume
hzbrl   1
N Yesterday at 6:32 PM by hzbrl
Source: purple comet
A large sphere with radius 7 contains three smaller balls each with radius 3 . The three balls are each externally tangent to the other two balls and internally tangent to the large sphere. There are four right circular cones that can be inscribed in the large sphere in such a way that the bases of the cones are tangent to all three balls. Of these four cones, the one with the greatest volume has volume $n \pi$. Find $n$.
1 reply
hzbrl
Thursday at 9:56 AM
hzbrl
Yesterday at 6:32 PM
(n+1)2^n, (n+3)2^{n+2} not perfect squares for the same n
parmenides51   3
N Yesterday at 6:31 PM by AylyGayypow009
Source: Greece JBMO TST 2015 p3
Prove that there is not a positive integer $n$ such that numbers $(n+1)2^n, (n+3)2^{n+2}$ are both perfect squares.
3 replies
parmenides51
Apr 29, 2019
AylyGayypow009
Yesterday at 6:31 PM
IMO Shortlist 2010 - Problem N2
Amir Hossein   29
N Apr 8, 2025 by cursed_tangent1434
Find all pairs $(m,n)$ of nonnegative integers for which \[m^2 + 2 \cdot 3^n = m\left(2^{n+1} - 1\right).\]

Proposed by Angelo Di Pasquale, Australia
29 replies
Amir Hossein
Jul 17, 2011
cursed_tangent1434
Apr 8, 2025
IMO Shortlist 2010 - Problem N2
G H J
The post below has been deleted. Click to close.
This post has been deleted. Click here to see post.
Amir Hossein
5452 posts
#1 • 5 Y
Y by ahmedosama, Davi-8191, Adventure10, DEKT, and 1 other user
Find all pairs $(m,n)$ of nonnegative integers for which \[m^2 + 2 \cdot 3^n = m\left(2^{n+1} - 1\right).\]

Proposed by Angelo Di Pasquale, Australia
This post has been edited 1 time. Last edited by Amir Hossein, Jul 17, 2011, 2:48 AM
Z K Y
The post below has been deleted. Click to close.
This post has been deleted. Click here to see post.
Love_Math1994
200 posts
#2 • 4 Y
Y by Adventure10, Mango247, and 2 other users
Some discuss in here
http://www.artofproblemsolving.com/Forum/viewtopic.php?f=56&t=409486
[Moderator edit: And here. :D]
Z K Y
The post below has been deleted. Click to close.
This post has been deleted. Click here to see post.
oneplusone
1459 posts
#3 • 19 Y
Y by Binomial-theorem, Andy Loo, Amir Hossein, hEatLove, zarengold, joybangla, StanleyST, quangminhltv99, v_Enhance, Siddharth03, guptaamitu1, SADAT, mathmax12, Adventure10, Mango247, farhad.fritl, MS_asdfgzxcvb, and 2 other users
I have another competely different solution.

Taking mod $m$, we get $m\mid 2\times 3^n$. Since if $(m,n)$ is a solution, $\left(\frac{2\times 3^n}{m},n\right)$ is also a solution, so WLOG assume $m=3^a$. Then we have $3^a+2\times 3^{n-a}=2^{n+1}-1$. If either $a<3$ or $n-a<3$, it is easy to check that $(a,n)=(2,3),(2,5)$ are the only solutions by simple bounding. So now assume both $a$ and $n-a$ are at least 3. Taking mod 27, we get $18\mid n+1$. If $a\leq n-a$, then $3^a+2\times 3^{n-a}=3^a(1+2\times 3^{n-2a})$. Taking mod 19, we get $19\mid RHS$, so $19\mid 1+2\times 3^{n-2a}\implies n-2a\equiv 2\pmod{18}$. Taking mod 7, $7\mid 1+2\times 3^{n-2a}\implies n-2a\equiv 1\pmod{6}$. Contradiction! Now if $a>n-a$, $3^a+2\times 3^{n-a}=3^{n-a}(3^{2a-n}+2)$. Similarly, taking mod 19, we get $19\mid 3^{2a-n}+2\implies 2a-n\equiv 16\pmod{18}$ and taking mod 7 gives $2a-n\equiv 5\pmod 6$. Contradiction! So $(2,3),(2,5)$ are the only solutions for $(a,n)$, so $(m,n)=(9,3),(6,3),(9,5),(54,5)$.
Z K Y
The post below has been deleted. Click to close.
This post has been deleted. Click here to see post.
Lyub4o
265 posts
#4 • 2 Y
Y by Adventure10, Mango247
Can someone explain again the part with the moduls,please?
Z K Y
The post below has been deleted. Click to close.
This post has been deleted. Click here to see post.
ZODIACORACLE
7 posts
#5 • 4 Y
Y by AlastorMoody, F10tothepowerof34, Adventure10, Mango247
Similar to oneplusone's solution, WLOG assume $m=3^t$. Therefore we get $3^t+2\cdot 3^{n-t}=2^{n+1}-1$.
Suppose $2<t$ and $2<n-t$, we get$27|3^t+2\cdot 3^{n-t}=2^{n+1}-1$.So $18|n+1$, and $73|2^18-1|2^{n+1}-1$.
But $3^i\equiv \pm(1,3,9,27,8,24)\small(mod73)$, and it is easy to check that there is no $i,j\in \mathbb{Z}$ such that $3^i+2\cdot 3^j\equiv 0\small(mod73)$.That's contradiction and we get $t=0,1,2$ or $n-t=0,1,2$.
After some research with inequalities, we get $(n,t)=(3,2),(5,2)$. Therefore$(m,n)=(6,3),(9,3),(9,5),(54,5)$.
Z K Y
The post below has been deleted. Click to close.
This post has been deleted. Click here to see post.
Wolowizard
617 posts
#6 • 3 Y
Y by Adventure10, Mango247, Deadline
We have $m|2\cdot 3^n$ so $m=3^k$ or $m=2\cdot 3^k$.
First case $m=3^k$.
$3^{2k}+2\cdot3^n=3^k(2^{n+1}-1)$
$3^k+2 \cdot 3^{n-k}=(2^{n+1}-1)$
Taking $mod 4$ we have that $k=2m$ is even and $n+1$ is even.
We have $3^{min(k,n-k)}|2^n+1$ so
$min(k,n-k)\le V_3(n+1)+1$ so we have that $2^{n+1}-1=3^k+23^{n-k}>3^{max(n,n-k))}=3^{n-V_3(n+1)-1)}>3^{n-log_3(n+1)}=3^{n}/(n+1)$ which implies //
$2^{n+1}(n+1)>3^{n}$ so $n+1\le 10$.
If $m=23^k$:
$23^k+3^{n-k}=2^{n+1}-1$ so we have
$min(k,n-k)\le V_3(n+1)+1<log_3(n+1)+1$ but
$2^{n+1}>2^{n+1}-1=23^k+3^{n-k}>3^{max(k,n-k)}>3^{n-min(k,n-k)}>3^{n-log_3(n+1)}=3^{n}/(n+1)$ so $n+1\le 10$.
Working out each $n$ we get $(n,m)=(3,6),(3,9),(5,9),(5,54)$.
This post has been edited 1 time. Last edited by Wolowizard, Dec 31, 2016, 7:55 PM
Z K Y
The post below has been deleted. Click to close.
This post has been deleted. Click here to see post.
v_Enhance
6877 posts
#7 • 13 Y
Y by Wizard_32, Alireza_Amiri, H.M-Deadline, Limerent, srijonrick, v4913, guptaamitu1, CoderNp, Epsilon-, Adventure10, Mango247, NicoN9, Funcshun840
The answer is $(m,n) \in \left\{ (9,3), (6,3), (9,5), (54,5) \right\}$, which work.

As the equation is a quadratic in $m$ for any fixed $n$, we will show that $n \notin \{3,5\}$ do not work.
Taking the discriminant, it's equivalent to solving \[ t^2 = \left( 2^{n+1}-1 \right)^2 - 4 \cdot 2 \cdot 3^n \iff 8 \cdot 3^n = \left( 2^{n+1}-1 \right)^2 - t^2. \]The right-hand side factors as a difference of squares, where both factors sum to $2(2^{n+1}-1)$. In particular, these factors have the same parity, and thus must be of the form $2 \cdot 3^a$ and $4 \cdot 3^b$, for nonnegative integers $a$, $b$ with $a+b=n$. Rearranging, we obtain \[ 2^{a+b+1} - 1 = 3^a + 2 \cdot 3^b. \]We have solutions $(a,b) = (2,1)$ and $(a,b) = (2,3)$, which corresponds to $n=3$ and $n=5$ above. Manual inspection reveals no other solutions with $\min(a,b) \le 2$, so we will suppose $\min(a,b) > 3$ and derive a contradiction.
Taking modulo $8$, we find $a$ is even and $b$ is odd (hence unequal). The idea is that the exponent of $3$ is much too large in the above equation. Indeed, \begin{align*} \min(a,b) &\le \nu_3(3^a + 2 \cdot 3^b) = \nu_3\left( 2^{a+b+1}-1 \right) \\ &= \nu_3\left( 4^{\frac{a+b+1}{2}} - 1  \right) = 1 + \nu_3\left( \frac{a+b+1}{2} \right) \\ &\le 1 + \log_3\left( \frac{a+b+1}{2} \right) \le 1 + \log_3(\max\{a,b\}) \\ \implies \max(a,b) &\ge 3^{\min(a,b)-1}. \end{align*}Intuitively, this is way too big. We write \begin{align*} 2^{\max(a,b) + \min(a,b) + 1} &= 2^{a+b+1} = 1 + 3^a + 2 \cdot 3^b > 3^{\max(a,b)} \\ \implies 2^{\min(a,b)+1} &> \left( \frac 32 \right)^{\max(a,b)} \ge \left( \frac 32 \right)^{3^{\min(a,b)-1}} \end{align*}which is false for $\min(a,b) \ge 3$, as desired.
This post has been edited 3 times. Last edited by v_Enhance, Mar 12, 2024, 1:37 AM
Z K Y
The post below has been deleted. Click to close.
This post has been deleted. Click here to see post.
cjfev4
10 posts
#8 • 2 Y
Y by Adventure10, Mango247
oneplusone wrote:
I have another competely different solution.

Taking mod $m$, we get $m\mid 2\times 3^n$. Since if $(m,n)$ is a solution, $\left(\frac{2\times 3^n}{m},n\right)$ is also a solution, so WLOG assume $m=3^a$. Then we have $3^a+2\times 3^{n-a}=2^{n+1}-1$. If either $a<3$ or $n-a<3$, it is easy to check that $(a,n)=(2,3),(2,5)$ are the only solutions by simple bounding. So now assume both $a$ and $n-a$ are at least 3. Taking mod 27, we get $18\mid n+1$. If $a\leq n-a$, then $3^a+2\times 3^{n-a}=3^a(1+2\times 3^{n-2a})$. Taking mod 19, we get $19\mid RHS$, so $19\mid 1+2\times 3^{n-2a}\implies n-2a\equiv 2\pmod{18}$. Taking mod 7, $7\mid 1+2\times 3^{n-2a}\implies n-2a\equiv 1\pmod{6}$. Contradiction! Now if $a>n-a$, $3^a+2\times 3^{n-a}=3^{n-a}(3^{2a-n}+2)$. Similarly, taking mod 19, we get $19\mid 3^{2a-n}+2\implies 2a-n\equiv 16\pmod{18}$ and taking mod 7 gives $2a-n\equiv 5\pmod 6$. Contradiction! So $(2,3),(2,5)$ are the only solutions for $(a,n)$, so $(m,n)=(9,3),(6,3),(9,5),(54,5)$.

why did you just assume $m=3^a$ ??? How about $m=2.3^a$ ??? Because it said that $m\mid 2\times 3^n$
Please, I don't understand. Thanks
Z K Y
The post below has been deleted. Click to close.
This post has been deleted. Click here to see post.
Pungpondopas
13 posts
#9 • 1 Y
Y by Adventure10
cjfev4 wrote:
oneplusone wrote:
I have another competely different solution.

Taking mod $m$, we get $m\mid 2\times 3^n$. Since if $(m,n)$ is a solution, $\left(\frac{2\times 3^n}{m},n\right)$ is also a solution, so WLOG assume $m=3^a$. Then we have $3^a+2\times 3^{n-a}=2^{n+1}-1$. If either $a<3$ or $n-a<3$, it is easy to check that $(a,n)=(2,3),(2,5)$ are the only solutions by simple bounding. So now assume both $a$ and $n-a$ are at least 3. Taking mod 27, we get $18\mid n+1$. If $a\leq n-a$, then $3^a+2\times 3^{n-a}=3^a(1+2\times 3^{n-2a})$. Taking mod 19, we get $19\mid RHS$, so $19\mid 1+2\times 3^{n-2a}\implies n-2a\equiv 2\pmod{18}$. Taking mod 7, $7\mid 1+2\times 3^{n-2a}\implies n-2a\equiv 1\pmod{6}$. Contradiction! Now if $a>n-a$, $3^a+2\times 3^{n-a}=3^{n-a}(3^{2a-n}+2)$. Similarly, taking mod 19, we get $19\mid 3^{2a-n}+2\implies 2a-n\equiv 16\pmod{18}$ and taking mod 7 gives $2a-n\equiv 5\pmod 6$. Contradiction! So $(2,3),(2,5)$ are the only solutions for $(a,n)$, so $(m,n)=(9,3),(6,3),(9,5),(54,5)$.

why did you just assume $m=3^a$ ??? How about $m=2.3^a$ ??? Because it said that $m\mid 2\times 3^n$
Please, I don't understand. Thanks

Because if we get $m=3^a$ ———> $(m,n)$ is a solution,we will get $m’=\frac{2\times 3^n}{m}$ and $2|m’$ ,$(m’,n)$ is also a solution
Z K Y
The post below has been deleted. Click to close.
This post has been deleted. Click here to see post.
AlastorMoody
2125 posts
#10 • 4 Y
Y by GeoMetrix, lilavati_2005, Aryan-23, Adventure10
Solution
Z K Y
The post below has been deleted. Click to close.
This post has been deleted. Click here to see post.
pad
1671 posts
#11 • 1 Y
Y by farhad.fritl
We have
\[ m^2-(2^{n+1}-1)m + 2\cdot 3^n = 0. \]So
\[ (2^{n+1}-1)^2 - 8\cdot 3^n = k^2 \]for some $k$. Then
\[ 8\cdot 3^n = (2^{n+1}-1-k)(2^{n+1}-1+k). \]So for some $a,b$ with $a+b=n$, we have
\[ \{ 2^{n+1}-1-k, \ 2^{n+1}-1 + k \} = \{2\cdot 3^a,\ 4\cdot 3^b \}.\]We can't have $3^a, 8\cdot 3^b$ because then the sum of the two would be odd, but it is clearly even. Now, adding gives
\[ 2^{n+2} - 2 = 2\cdot 3^a + 4\cdot 3^b \implies 2^{n+1} - 1 = 3^a + 2\cdot 3^b. \]mod 8 gives $a$ is even and $b$ is odd, so $n=a+b$ is odd too. Hence $n+1$ is even, so
\[ \nu_3(2^{n+1}-1) = \nu_3(2^{n+1} - (-1)^{n+1}) = 1 + \nu_3(n+1) \ge \min(a,b). \]Then
\[ 1+\nu_3(a+b+1) \ge \min(a,b) \implies a+b+1 \ge 3^{\min(a,b)-1}. \]This is a strong bound. We also know $2^{a+b+1} - 1 = 3^a + 2\cdot 3^b$. Intuitively, the RHS is way too big, using the above bound. Suppose $b>a$, in order to make the RHS bigger. We will still use size to narrow down the possibilities. So $\min(a,b)=a, \max(a,b)=b$. We know $a+b+1 \ge 3^{a-1}$, so $b\ge 3^{a-1}-a-1$. Then
\[ 2^{a+b+1} = 1+3^a+2\cdot 3^b > 3^b \implies 2^{a+1} > 1.5^b \ge 1.5^{3^{a-1}-a-1}. \]If $a\ge 4$, this is not true. So $a\le 3$. Now manual case check. It turns out the answer is s $(m,n) =(9,3), (6,3), (9,5), (54,5)$.
Z K Y
The post below has been deleted. Click to close.
This post has been deleted. Click here to see post.
VulcanForge
626 posts
#12
Y by
View the equation as a quadratic in $m$; then by Vieta there must be integers $x,y$ with $xy=2 \cdot 3^n$ and $x+y=2^{n+1}-1$. WLOG let $x$ be even, and let $x=2 \cdot 3^a, y=3^b$; then we want to solve the equation $$2 \cdot 3^a+3^b = 2^{a+b+1}-1$$. Take $\nu_3$ of both sides to get by LTE that $$\text{min}(a,b) \leq \nu_3 \left( 4^{\frac{a+b+1}{2}}-1 \right) = 1+\nu_3(a+b+1)<1+\log_3(a+b+1)$$which implies that $$a+b+1 > 3^{\text{min}(a,b)-1}$$. For convenience, let $\text{min}(a,b)=t$ and $\text{max}(a,b)=T$. Now note that $2^{a+b+1}>3^{T}$ so from the previous centered equation we have $$2^{t+1}>1.5^T=1.5^{a+b-t}>1.5^{3^{t-1}-1-t}$$which is only true when $t \leq 3$. From here it is easy to manually check that the only solutions are $$(m,n)=\boxed{(6,3),(9,3),(9,5),(54,5)}$$.
This post has been edited 1 time. Last edited by VulcanForge, Jun 4, 2020, 6:39 PM
Z K Y
The post below has been deleted. Click to close.
This post has been deleted. Click here to see post.
lazizbek42
548 posts
#13
Y by
Amir Hossein wrote:
Find all pairs $(m,n)$ of nonnegative integers for which \[m^2 + 2 \cdot 3^n = m\left(2^{n+1} - 1\right).\]
Proposed by Angelo Di Pasquale, Australia
$$m|2\cdot 3^n \implies m=3^k ; m=2\cdot 3^k$$after several definitions, both of these cases are sufficient to solve the following equation.
$$2\cdot 3^a + 3^b=2^{a+b+1}-1$$1)$a=b$ in which case it is not difficult to solve.(mod8).
2)$a>b \implies  3^b|2^{a+b+1}-1 \implies v_3(a+b+1) \ge b-1 \implies a+b+1 \ge 3^{ v_3(a+b+1)} \ge 3^{b-1}$
$$2^{a+b+1} > 2 \cdot 3^a$$$$(\frac{3}{2})^{2b} > 2^b > (\frac{3}{2})^a \implies 2b>a >3^{b-1}-b-1$$this leads to a boundary of $b$, and it is easy to see the rest, just as $b> a$.
Z K Y
The post below has been deleted. Click to close.
This post has been deleted. Click here to see post.
asdf334
7585 posts
#14
Y by
Note that $m\mid 2\cdot 3^n$. Also, if $m$ is a solution for a fixed $n$ then $\frac{2\cdot 3^n}{m}$ is also a solution. Therefore we may WLOG $m=2\cdot 3^k$. Note that we then get $$2^{n+1}-1=3^{n-k}+2\cdot 3^k.$$Now if $k=0$ or $k=n$ we can check that no solutions exist which implies that $2^{n+1}\equiv 1\pmod 3$ or that $n$ is odd. Then we have
\[\nu_3(2^{n+1}-(-1)^{n+1})=1+\nu_3(n+1)=\text{min}(k,n-k)\]so now we split into cases. If $k> \frac{n}{2}$ then we have $\nu_3(n+1)=n-k-1$ which implies that $3^{n-k-1}\le n+1$ or that $3^k\ge \frac{3^{n-1}}{n+1}$ which means that
\[2^{n+1}-1=3^{n-k}+2\cdot 3^k>\frac{2\cdot 3^{n-1}}{n+1}\]so that $n\le 8$. Then $n=1,3,5,7$ which gives $k=0,2,3,6$. Only $(k,n)=(3,5)$ works and gives the solutions $(m,n)=(54,5),(9,5)$. Now suppose that $k<\frac{n}{2}$. Then we have $\nu_3(n+1)=k-1$ so we get $3^{k-1}\le n+1$ and $3^{n-k}\ge \frac{3^{n-1}}{n+1}$. Bounding, we get $n\le 10$, so that $n=1,3,5,7,9$ and $k=1,1,2,1,1$. Only $(k,n)=(1,3)$ works so we get the final two solutions $(m,n)=(6,3),(9,3)$. We are done. $\blacksquare$
Z K Y
The post below has been deleted. Click to close.
This post has been deleted. Click here to see post.
awesomeming327.
1717 posts
#15
Y by
We have $m(2^{n+1}-1-m)=2\cdot 3^n$, so $\{m,2^{n+1}-1-m\}=\{x,y\}$ where $x=2\cdot 3^i$ and $y=3^j$ and $x+y=2^{i+j+1}-1.$ Consider $\nu_3(x+y)=\min(i,j).$

If $2\nmid n+1=i+j+1$ the RHS isn't divisible by $3$, so $i=0$ or $j=0.$ Thus, $(x,y)=(2,3^n)$ or $(1,2\cdot 3^n).$ One of these equals $m$ so pluggin $m=1,2,3^n,2\cdot 3^{n}.$ $m=1$ gives no solutions by bounding, For $m=2$, take mod $3$ to get $0\equiv 2^{n+2}\pmod 3$, absurd. For $m=3^n$ does not work again because we get $3^n+3=2^{n+1}$ so $3\mid 2^{n+1}.$ For $m=2\cdot 3^{n}$ does not work by bounding. In summary, no solutions.

Now, we have $2\mid n+1$ so \[\min(i,j)=\nu_3(4^{\frac{n+1}{2}}-1)=1+\nu_3(\frac{n+1}{2})=1+\nu_3(i+j+1)\le 1+\log_3(i+j+1)\]Thus, $i+j+1\ge 3^{\min(i,j)-1}$

Now, $2^{n+1}\ge 3^{\max(i,j)}$ so $2^{\min(i,j)+1}\ge 1.5^{\max(i,j)}=1.5^{i+j-\min(i,j)}>1.5^{3^{\min(i,j)-1}-1-\min(i,j)}.$ Suppose $\min(i,j)>3$ then this is a contradiction.

Now, check $\min(i,j)=1,2,3$ to get our solutions.
Z K Y
The post below has been deleted. Click to close.
This post has been deleted. Click here to see post.
Ibrahim_K
62 posts
#17
Y by
Nice problem ... My solution is quite long so I will share in short way with some parts to solve it.
Part 1
Taking mod $m\implies$ $m\mid 2\times 3^n$
Part 2
Let $m=3^a$
Part 3
We show that if $a>2$ and $n-a>2\implies$ $n\equiv -1\pmod{18}$ (by mod$27$ )
Part 4
Then $19\mid 2\times 3^a + 3^{n-a}$
Part 5
Divide into two cases $n>2a$ or $n<2a$
Part 6
Prove that there is not any solution in both cases(from mod$19$ and mod$2$)
Part 7
Thus $a>2$ and $n-a>2$ can not be true at same time , Work on $1) a=1 , 2) a=2 , 3) a>2,n-a<2$
Part 8
Find $m=9 , n=3$ and $m=9 , n=5$
Part 9
Now let $m= 2\times 3^a$ and do same things above
Part 10
Find $m=6 , n=3$ and $m=54 , n=5$

So we are done.
This post has been edited 1 time. Last edited by Ibrahim_K, Dec 13, 2022, 6:25 PM
Z K Y
The post below has been deleted. Click to close.
This post has been deleted. Click here to see post.
BobsonJoe
811 posts
#18
Y by
Take the equation as a quadratic in $m$. Since the discriminant is a square, we have
\[ (2^{n+1} - 1)^2 - 8\cdot 3^n \implies 8\cdot 3^n = (2^{n+1} - 1 + k)(2^{n+1} - 1 - k)\]Note that the factors must be of the form $2\cdot 3^x$ and $4\cdot 3^y$, with $x + y = n$. Allowing $k$ to be negative, we may assume WLOG that the first factor is $2\cdot 3^x$ and the second one is $4\cdot 3^y$. Hence,
\[k = 3^x - 2\cdot 3^y \text{ and } 2^{n+1} - 1 = 3^x + 2\cdot 3^y\]We now bound $x$ and $y$. We have
\[3^x < 2^{n+1} \implies 3^y > \frac{3^n}{2^{n+1}} \text{ and } 3^y < 2^n \implies 3^x> \frac{3^n}{2^n}\]Hence,
\[\frac{3^n}{2^{n+1}} < 3^{\min (x, y)} < 3^{\nu_3 (2^{n+1} - 1)} = 3^{1 + \nu_3 ((n+1)/2)} \le 3\cdot \frac{n+1}{2}\]This gives $n\in \{1, 3, 5, 7\}$. We get the solutions $\boxed{(m, n) = (6, 3), (9, 3), (9, 5), (54, 5)}$.
Z K Y
The post below has been deleted. Click to close.
This post has been deleted. Click here to see post.
Marinchoo
407 posts
#19 • 1 Y
Y by VicKmath7
Consider the equation as a quadratic in $m$, that is, let $P_{n}(x)=x^2-x(2^{n+1}-1)+2\cdot 3^{n}$. Then, if $(m,n)$ is a solution, $m$ is a root of $P_{n}(x)=0$ and so the second root must be $2^{n+1}-1-m\in\mathbb{Z}$ which is actually a positive integer as $2^{n+1}-1-m = \frac{2\cdot 3^{n}}{m}>0$ by Vieta's formulas. Hence if $(m,n)$ is a solution, then $P_{n}(x)$ has positive integer roots $m_{1}, m_{2}$. Note that $m_{1}+m_{2}=2^{n+1}-1$ and $m_{1}m_{2}=2\cdot 3^{n}$. A direct check shows that $m=1$ and $m=2$ lead to no solutions, which implies that $3\mid m_{1}$ and $3\mid m_{2}$, so $3\mid m_{1}+m_{2} = 2^{n+1}-1$, so $n+1$ is even. WLOG let $m_{1}\leq m_{2}$. Now $m_{1}m_{2}=2\cdot 3^{n}\Longrightarrow \nu_{3}(m_{1})\leq \nu_{3}(m_{2})$ whence
\[\nu_{3}(m_{1})\leq \nu_{3}(m_{1}+m_{2}) = \nu_{3}(2^{n+1}-1)=\nu_{3}(4^{\frac{n+1}{2}}-1)=1+\nu_{3}\left(\frac{n+1}{2}\right)\leq 1+\log_{3}\left(\frac{n+1}{2}\right) \Longrightarrow m_{1}\leq \frac{3(n+1)}{2}\]by LTE. Note that $f(x)=x+\frac{c}{x}$ is decreasing on the interval $(0,\sqrt{c}]$ where $c$ is any positive real number as $f'(x)=1-\frac{c}{x^2}<0$. Now assume that $n\geq 6$. Clearly we have that $\sqrt{2\cdot 3^{n}}>\frac{3(n+1)}{2}$, so we have that:
\[2^{n+1}-1 = m_{1}+m_{2} = m_{1}+\frac{2\cdot 3^{n}}{m_{1}} \geq \frac{3(n+1)}{2}+\frac{4\cdot 3^{n-1}}{n+1} \Longrightarrow (n+1)2^{n+2}-2n-2-3(n+1)^2-8\cdot 3^{n-1}\geq 0\]This is a contradiction as the last inequality doesn't hold for $n\geq 6$, here's a proof This implies that $n\leq 5$ and we're left to check the remaining cases. In the beginning, we mentioned that $n$ must be odd, so $n\in\{1,3,5\}$. A direct check shows that $n=1$ doesn't lead to a solution whereas $n=3$ and $n=5$ lead to the solutions $(m,n)=(6,3), (9,3), (9,5), (54,5)$.
Z K Y
The post below has been deleted. Click to close.
This post has been deleted. Click here to see post.
HamstPan38825
8862 posts
#20
Y by
Note that $m \mid 2 \cdot 3^n$.

Assume first that $m = 2 \cdot 3^k$ for some $k \leq n$. Then, the equation simplifies to $$2 \cdot 3^{2k} + 3^n = 3^k(2^{n+1} - 1).$$We take $\nu_3$ of both sides.
  • If $\nu_3(2^{n+1} - 1) = 0$, then we must have $n = k$. This reduces to $2 \cdot 3^n + 1 = 2^{n+1} - 1$, which is obviously impossible for size issues.
  • Otherwise, $n$ must be odd, and set $r = \nu_3(2^{n+1} - 1) = 1 + \nu_3\left(\frac{n+1}2\right) \leq 1+\log_3(n+1) - \log_3(2)$. Then if $r = n-k$, then $$2 \cdot 3^k + 3^{n-k} = 2^{n+1} - 1.$$But on the other hand $$n-k \leq \log_3(n+1) + 1 \iff n+1 \geq 3^{n-k-1}.$$But this implies $3^k \geq \frac{3^{n-1}}{n+1}$, or $$\frac{3^{n-1}}{n+1} \leq 2^{n+1} - 1 \iff n \leq 10.$$Finite case check yields $(6, 3)$ and $(54, 5)$. The other case works out in the exact same way by symmetry.
Now, if $m =3^k$, notice that the transformation $m \to \frac{2 \cdot 3^n}m$ bijects between solutions with $m = 2 \cdot 3^k$ and $m = 3^k$. Thus, the other two solutions are $(9, 5)$ and $(9, 3)$.
Z K Y
The post below has been deleted. Click to close.
This post has been deleted. Click here to see post.
bobthegod78
2982 posts
#21
Y by
It is easy to see $m\mid 2\cdot 3^n$, and if $\left(n, m\right)$ is a solution, then so is $\left( n, \frac{2\cdot 3^n}{m}\right)$, WLOG, $m=2\cdot 3^k$. We can rewrite the equation is $2\cdot 3^k + 3^{n-k} = 2^{n+1}-1$. If $k<3$ or $n-k<3$, we can easily check that $(n,m) = (5,9), (5,54), (3,9), (3,6)$ are the only solutions. Taking mod 27, we have $n\equiv 17 \pmod{18}$, but taking mod 73, we get $2\cdot 3^k + 3^{n-k} \equiv 0 \pmod{73},$ but $3^n \bmod{73}$ is either $3^5$ or $3^{11}$ (since the order of $3$ modulo $73$ is 12), and it is easy to check none of these yield solutions.

Motivation: I found some primes that divided $2^{18}-1$ and $73$ was interesting, since the order of 3 was also nice. From that it was fairly obvious.
This post has been edited 1 time. Last edited by bobthegod78, May 17, 2023, 11:19 PM
Z K Y
The post below has been deleted. Click to close.
This post has been deleted. Click here to see post.
mathmax12
6051 posts
#22
Y by
:skull: i only found the pairs (6,3),(9,3),(9,5) after a hour of dumb bashing
Z K Y
The post below has been deleted. Click to close.
This post has been deleted. Click here to see post.
huashiliao2020
1292 posts
#23
Y by
Same as most of the other ones, but this is a pretty nice problem.

First, mod m, we get $m\mid 3^n2$; in particular, we claim n is odd: If n were even note that 2^{n+1}-1 is not 0 mod 3. mod 3 we get $m^2\equiv m\implies m\equiv \{0,1\}\pmod 3$, so there are two cases.
1. m is 0 mod 3. Looking at $v_3, m=\{3^n,3^n2\}$. If $m=3^n,3^n(3^n+2)=3^n(2^{n+1}-1)$ has no solutions. If $m=3^n2, 3^n2(3^n2+1)=3^n2(2^{n+1}-1)$ also has no solutions.
2. m is 1 mod 3 from here we get a contradiction because we derive m=1, and checking, there are no sols.
We conclude that n is odd, and 2^n-1 is not divisible by 3. Now, check manually the cases for n<10.

Looking at the discriminant in the quadratic of m, we have $$(2^{n+1} - 1)^2 - 3^n8=k^2 \implies3^n8 = (2^{n+1} - 1 + k)(2^{n+1} - 1 - k)=ab\stackrel{\text{same mod 2,WLOG}}{\implies}(a,b)=(3^x2,3^y4),x + y = n\implies(k,2^{n+1}-1) = (3^x - 3^y2,3^x + 3^y2)\quad(1); k<2^{n+1}-1\implies 3^x2<2^{n+1}2-1\implies 3^x<2^{n+1}\implies 3^y > \frac{3^n}{2^{n+1}}\quad(2),3^y4<2^{n+1}\implies3^y < 2^{n-1} \implies 3^x> \frac{3^n}{2^{n+1}}\quad(3)$$$$\stackrel{(2),(3)}{\implies} \frac{3^n}{2^{n+1}} < 3^{\min (x, y)} \stackrel{(1)}{=} 3^{\nu_3(2^{n+1}-1)}=3^{\nu_3 ((2^n - 1)(2^n+1))} =3^{\nu_3(2^n+1)}= 3^{1 + \nu_3n} \le 3n\implies n\le10\implies(m, n) = \{(6, 3), (9, 3), (9, 5), (54, 5)\},$$as desired. $\blacksquare$

Remark. The part needing n is odd might be thought of as an unnecessary procedure, but it is well motivated when we get to the last few steps to need to derive it.
This post has been edited 2 times. Last edited by huashiliao2020, Aug 26, 2023, 5:41 PM
Z K Y
The post below has been deleted. Click to close.
This post has been deleted. Click here to see post.
Math4Life7
1703 posts
#24
Y by
We can take the equation $\pmod{m}$ to see that $2 \cdot 3^n \equiv 0 \pmod{m}$. We also know that $m$ must be odd because taking $v_2$, thus $m = 3^a$ or $m = 2\cdot 3^a$ for some positive integer $a \leq n$.

We first take care of $m = 3^a$ We can divide the equation by $3^a$ to get \[3^a + 2 \cdot 3^{n-a} = 2^{n+1} -1\]We can see that $v_3(3^a + 2 \cdot 3^n-a) \geq \min(a, n-a)$ (note the expression is not equal only when $2a =n$). We can see that the LHS is always divisible by $3$ (if $a = 0$ then the LHS is even which is not possible). This means that we can assume that $n+1$ is even. Thus we can see taking $v_3$ that we can see that we need \[\min(a, n-a) \leq 1+v_3\left(\frac{n+1}{2}\right)\]Notice that the first value of $n$ to attain $x$ on the LHS is $2 \cdot 3^{x-1} - 1$. This means that we need \[3^x + 2 \cdot 3^{2 \cdot 3^{x-1}-x - 1} \leq 2^{2 \cdot 3^{x-1}} - 1\]or \[3^{2 \cdot 3^{x-1}-x - 1} + 2 \cdot 3^x \leq 2^{2 \cdot 3^{x-1}} - 1\]The first equation obviously has size issues when $x \geq 3$. Checking those finite cases we get $(m, n) = (9, 5)$ Similarly we can see that the second equation has size issues when $x \geq 3$. Checking finite cases we get $(m, n) = (9, 3)$.

We can see that if $m = 2 \cdot 3^a$ then we can divide by $m$ to get $2 \cdot 3^a + 2 \cdot 3^{n-a} = 2^{n+1} - 1$ This solution set is actually isomorphic to the $m = 3^a$ solutions. Which give $\boxed{(m, n) = (9, 5), (9, 3), (54, 5), (6, 3)}$. $\blacksquare$
Z K Y
The post below has been deleted. Click to close.
This post has been deleted. Click here to see post.
Shreyasharma
682 posts
#25
Y by
What in the world is my solution.

We claim the only solutions are $(m,n) \in \left\{ (9,3), (6,3), (9,5), (54,5) \right\}$.

Taking modulo $m$ we have $m \mid 2 \cdot 3^n$. Thus let $m = 2 \cdot 3^b$ with $1 \leq b \leq n$. Then we have,
\begin{align*}
4 \cdot 3^{2b} + 2 \cdot 3^n &= 2^{n + 2}3^b - 2\cdot 3^b\\
2 \cdot 3^{2b} + 3^n &= 2^{n+1}3^b - 3^b\\
2 \cdot 3^b + 3^{n-b} &= 2^{n+1} - 1\\
3^{n-b} + 1 &= 2^{n+1} - 2 \cdot 3^b
\end{align*}Clearly $\nu_2(\text{RHS}) = 1$ and hence $n - b $ must be odd. Specifically $n$ and $b$ are of opposite parity. Reindex such that $n = a + b$ whence $a$ and $b$ must be of opposite parity. Then we have,
\begin{align*}
3^a + 1 &= 2^{a + b + 1} - 2 \cdot 3^b\\
2^{a+b+1} - 1 &= 3^a + 2 \cdot 3^ b
\end{align*}For $a, b \leq 3$ we can find solutions $n = 3$, $n = 5$. Assume that $\min(a, b) > 3$. Taking modulo $2 \cdot 3^3$ and modulo $3^3$ we find that $a \equiv 0 \pmod{18}$ and $b \equiv -1 \pmod{18}$. Finally taking modulo $127$ we find that,
\begin{align*}
0 &\equiv 3^a + 2 \cdot 3^b \pmod{127}\\
-3^a &\equiv 2 \cdot 3^b \pmod{127}\\
3^{a - b} &\equiv 125 \pmod{127}
\end{align*}Now we claim that there are no solutions. Indeed looking at the residues $3$ leaves for powers congruent to $1$ modulo $18$ we see, $3^{18k + 1} \in \{3, 12, 48, 65, 6, 24, 96\}$ hence we are done.
Z K Y
The post below has been deleted. Click to close.
This post has been deleted. Click here to see post.
AlanLG
241 posts
#26
Y by
Clearly $m\mid 2\cdot 3^n$

We are going to check the case when $m=2\cdot 3^\theta$ then
$$4\cdot 3^\theta+2\cdot 3^{n-\theta}=2^{n+1}-1$$But this fails by $\pmod 2$

So $m=3^k$ and the equation becomes
$$3^k+2\cdot 3^{n-k}=2^{n+1}-1$$We can easily check that nor $k=0, n=k$, then $n$ must be odd
Now check $\nu_3$ of the equation and note that
$$\min\{k,n-k\}=\nu_3(3^k+2\cdot 3^{n-k})=\nu_3\left(4^{\frac{n+1}{2}}-1\right)=1+\nu_3\left(\frac{n+1}{2}\right)\leq 1+\log_3\left(\frac{n+1}{2}\right)\leq 1+\log_3\left(\max\{k,n-k\} \right)$$so $\max\{n,n-k\} \geq 3^{\min\{k,n-k\}}$
But in the equation
$$2^{\max\{n,n-k\}+{\min\{k,n-k\}}}=3^k+2\cdot 3^{n-k}+1>3^{\max\{n,n-k\}}$$so $$2^{\min\{k,n-k\}+1}>\left(\frac{3}{2} \right)^{\max\{n,n-k\}}\geq \left(\frac{3}{2}\right)^{3^{\min\{k,n-k\}}} $$Which fails for ${\min\{k,n-k\}}\geq 3$ so we can manually check for ${\min\{k,n-k\}}=1,2,3$
Z K Y
The post below has been deleted. Click to close.
This post has been deleted. Click here to see post.
joshualiu315
2534 posts
#27
Y by
sketch
Z K Y
The post below has been deleted. Click to close.
This post has been deleted. Click here to see post.
shendrew7
795 posts
#28
Y by
We claim our only solutions are $\boxed{(9,3), (6,3), (9,5), (54,5)}$. Expressing the equation as a quadratic in $m$, we require
\[8 \cdot 3^n = (2^{n+1}-1)^2-x^2 = (2^{n+1}-1-x)(2^{n+1}-1+x)\]
for an integer $x$. Parity tells us these factors correspond to $(2 \cdot 3^y) \cdot (4 \cdot 3^{n-y}$. Modulo 8 further tells us $y \neq n-y$, so assume $y<n-y$ (where the other case is analogous) to give
\[2^{n+1}-1 = 2 \cdot 3^y + 3^{n-y} = 3^y \left(3^{n-2y}+2\right).\]
If $y$ or $n-y$ are at most 2, casework reveals the solutions above. Otherwise, $n+1$ must be even, so we use LTE to find
\[v_3(2^{n+1}-1) = v_3\left(\frac{n+1}{2}\right)+1 \ge y \implies n+1 \ge 2 \cdot 3^{y-1},\]
which is evidently too large if $y \ge 3$. (One way to see this is by noting the RHS of our equation grows faster than the left, so we can isolate $n$ and find an upper bound in terms of $y$ which excludes the interval we just proved.) $\blacksquare$
Z K Y
The post below has been deleted. Click to close.
This post has been deleted. Click here to see post.
ezpotd
1266 posts
#29
Y by
Quadratic formula forces that $(2^{n +1} - 1)^2 - 8 \cdot 3^n$ is a perfect square, using difference of squares we get $(2^{n + 1} - 1 - k )\cdot (2^{n + 1} - 1 + k) = 8 \cdot 3^n$, so one of the two terms on the left side is $2 \cdot 3^a$, the other is $4 \cdot 3^b$, so we get $2 \cdot 3^a + 4 \cdot 3^b = 2(2^{n+ 1} - 1)$, so we get $3^a + 2 \cdot 3^b = 2^{n + 1} - 1$, with $a + b = n$. If $n$ is even, the right side is not divisible by $3$, so one of $a,b$ is $0$. If $a = 0$, we get $3^n = 2^n - 1$, which obviously has no solutions. If $b = 0$, we get $3^n = 2^{n + 1} - 3$., which also has no solutions by size. Thus $n$ is odd, so $\nu_3(2^{n + 1} - 1) = 1 + \nu_3(\frac{n +1}{2})$, and $\nu_3(3^a + 2 \cdot 3^b) \ge \min(a,b)$, so $1 + \log_3 n \ge 1 + \log_3  \frac{n + 1}{2} \ge 1 + \nu_3(\frac{n +1}{2}) = \nu_3(3^a + 2 \cdot 3^b) \ge min(a, n - a)$. Thus $\max(a,b) \ge n - 1- \log_3 n$. Thus we can bound $2^{n + 1 } - 1 =3^a + 2 \cdot 3^b \le 3 \cdot 3^{\max(a,b)} \le 3^{n - \log_3 n} = \frac{3^n}{n}$. Thus, we can start by eliminating all $n$ for which $3^{n} > (2n)2^n$, or equivalently $(\frac{3}{2})^n \ge 2n$. We can observe $\frac{3^7}{2^7} = \frac{2187}{128} > 14$, then taking derivatives of both sides, we see $(\frac 32)^n \ln \frac 32 > 2$ for $n \ge 7$ as $\ln 32 < 3$ as $(1.5)^3 > 3 > e$.

We then manually solve the cases for $n$ between $0$ and $6$. We already forced $n$ odd, so it remains to check $n =  1,3,5$. Checking $n = 1$ gives $m^2 -3n + 6$, which has no solutions, checking $n = 3$ gives $m^2 + 54 = 15m$, which resolves to $(m - 6)(m-  9) =0$, so we have solutions $(6,3), (9,3)$, both of which clearly work. Checking $n = 5$ gives $m^2 + 486 = 63$, which resolves to $(m - 54)(m - 9) = 0$, so we have solutions $(54, 5), (9,5)$, both of which clearly work. We have exhausted all cases, so we are done.
Z K Y
The post below has been deleted. Click to close.
This post has been deleted. Click here to see post.
megarnie
5606 posts
#30
Y by
Solved a while ago but forgot to post

The only solutions are \[(m,n) = (6,3),  (9,3), (9,5), (54,5), \]which clearly work. Now we show they are the only ones

If $n=3$, then $m^2  - 15m + 54 = 0\implies m\in \{6,9\}$.

If $n=5$, then $m^2 - 63m + 486 = 0\implies m\in \{9,54\}$.

Thus it suffices to show that $n\in \{3,5\}$.


Taking the equation as a quadratic in $m$, by taking the discriminant, we find \[k^2 = (2^{n+1} - 1)^2 - 8\cdot 3^n \]for some nonnegative integer $k$, so \[8\cdot 3^n = (2^{n+1} - k - 1)(2^{n+1} + k - 1)\]
Since the two terms in the product are of the same parity, they are equal to $2\cdot 3^a$ and $4\cdot 3^b$ in some order. So $n=a+b$. The equation is now equivalent to \[2\cdot 3^a + 4\cdot 3^b = 2^{a+b+2}- 1,\]so \[3^a + 2\cdot 3^b = 2^{a+b+1}  - 1\]
If $a=0$, then $2\cdot 3^b + 1 = 2^{b+1} - 1$, absurd. If $b=0$, then $3^a + 2 = 2^{a+1}  - 1$, also absurd.

Now taking $\pmod 8$ gives $a$ even and $b$ odd (so $a\ne b$).

Since $a+b$ is odd, we have $\nu_3(2^{a+b+1} - 1) = \nu_3(a+b+1) + 1$ (this can be found by factoring and LTE).

Let $\min(a,b)= c$ and $\max(a,b) = M$.

Taking $\nu_3$ of the equation, we get \[c-1 = \nu_3(a+b+1) .\]Since $a+b+1$ is even, we have \[a+b+1 \ge 2\cdot 3^{c - 1}\]
On the other hand, we get \[2^{a+b+1} > 3^{M}\]so \begin{align*}
2^{c + 1}  \\
> (1.5)^{M}\\
>M \\
\end{align*}
We have \[2M\ge a+b+1,\]so \[2^{c+1} > 3^{c- 1} \implies c\le 4\]
Also \[2^{M + 5} \le  2^{c + M + 1} >3^M, \]so $M\le 8$.


If $c=4$, then since $b$ is odd, we have $a=4$. So \[81 + 2\cdot 3^b = 2^{b+5} - 1\]Its easy to check that no odd $b\le 8$ satisfies this.

If $c=3$, then since $a$ is even, we have $b=3$. So \[3^a + 54 = 2^{a+4} - 1\]It's easy to check that all even $a\le 8$ do not satisfy this.

If $c=2$, then since $b$ is odd, we have $a=2$. So \[9 + 2\cdot 3^b = 2^{b+3} - 1\]Here we can check that the only possibility is $b=3$, since $b>c = 2$. This gives $n=5$.

If $c=1$, then since $a$ is even, we have $b=1$. So \[3^a + 6 = 2^{a+2} - 1\]Here we must have $a=2$, which gives $n=3$.

Thus $n\in \{3,5\}$.
Z K Y
The post below has been deleted. Click to close.
This post has been deleted. Click here to see post.
cursed_tangent1434
624 posts
#31
Y by
Pretty similar to most bounding solutions. Extremely tricky to not fakesolve. We claim that the only pairs of solutions $(m,n)$ are $(6,3)$ , $(9,3)$ , $(9,5)$ and $(5,54)$ which can easily be seen to work. We now work on showing that they are the only ones.

We view the given equation as a quadratic in $m$ and look at the discriminant, which must be a perfect square. Note,
\[\Delta_m=(2^{n+1}-1)^2 - 4(2\cdot 3^n)= (2^{n+1}-1)^2-8\cdot 3^n \]Setting this expression equal to a perfect square and factoring we have that,
\[(2^{n+1}-1-x)(2^{n+1}-1+x) = 8 \cdot 3^n\]for some positive integer $x$. Clearly $x$ must be odd, and hence each of the above factors must be even. Thus, they are $2\cdot 3^a$ and $4 \cdot 3^b$ for a pair of non-negative integers $a,b$ such that $a+b=n$. Summing the expressions for each factor we have,
\begin{align*}
2^{n+2}-2&=4\cdot 3^a + 2 \cdot 3^b \\
2^{a+b+1}-1& = 2\cdot 3^a + 3^b
\end{align*}Say $a=0$ and $b>0$. Then the equation rewrites to
\[2^{n+1}=3^b +3\]which is a contradiction since the right-hand side is divisible by 3 but not the left. Similarly if $b=0$ and $a>0$ the equation reduces to
\[2^n=3^a-1\]to which the only solution is $n=2$ and $a=1$ and $n=3$ and $a=2$ by Mihaelescu's Theorem.

Thus, in what follows we consider $a,b>0$. Let $m = \min(a,b)$ and $M=\max(a,b)$.We now show the following.

Claim : For all valid pairs of solutions $(a,b)$ we must have $m \le 2$.

Proof : We note that $a,b >0$ means that $3$ divides the right-hand side and hence so must the left, implying that $a+b+1$ is even. Thus by Lifting the Exponent Lemma we have,
\[ \log_3\left(\frac{a+b+1}{2}\right) +1 \le \nu_3(3)+\nu_3\left(\frac{a+b+1}{2}\right) = \nu_3(2\cdot 3^a + 3^b) = \min(a,b)\]since $a \ne b$ as $a+b$ is odd. Now,
\begin{align*}
    \frac{a+b+1}{2} & \ge 3^{m-1}\\
    M+m +1 &\ge 2 \cdot 3^{m-1}\\
    M & \ge 2 \cdot 3^{m-1}-m-1
\end{align*}However,
\[ 2^{m+M+1}-1 =2^{a+b+1}-1  =2\cdot 3^a + 3^b > 3^M\]Thus,
\begin{align*}
    2^{m+M+1}-1 & > 3^M\\
    2^{m+1} & > \left(\frac{3}{2}\right)^M\\
    & > \left(\frac{3}{2}\right)^{2\cdot 3^{m-1}-m-1}\\
    & > \left(\frac{3}{2}\right)^{3^{m-1}+3^{m-2}}\\
    & = \left(\frac{27}{8}\right)^{2m-3}\\
    & > 3^{2m-3}\\
    & > 2^{m+1}
\end{align*}for all $m >2$, since $2\cdot 3^{m-1}-m-1 > 3^{m-1}+3^{m-2}$ for all $m \ge 3$ and $3^{2m-3}>2^{m+1}$ for all $m >2$. This is a clear contradiction and hence, $m \le 2$ as claimed.

We now deal with the simpler cases. First, consider the given equation $\pmod{8}$. Since $a,b >0$ we must have $a+b+1 \ge 3$ so,
\[-1 \equiv 2^{a+b+1}-1 = 2\cdot 3^a + 3^b\]Now, if $b$ were odd , $2\cdot 3^a + 3^b \equiv 2\cdot 1 +3 \equiv 5 \pmod{8}$ or $2\cdot 3^a+3^b \equiv 2 \cdot 3 +3 \equiv 1 \pmod{8}$ which is clearly impossible. Thus, $b$ must be even and since $a+b$ is odd, $a$ is in turn odd. Thus, we must have $a=1$ or $b=2$.

Case 1 : $a=1$. Then, the equation reduces to
\[2^{b+2}-1=6+3^b > 2^{b+2} > 2^{b+2}-1\]for all $b \ge 3$. Thus, we must have $b=2$. This implies that $n=3$.

Case 2 : $b=2$. Then, the equation reduces to
\[2^{a+3}-1 = 2\cdot 3^a+9 > 2^{a+3} > 2^{a+3}-1\]for all $a \ge 4$. Thus, we must have $a=1$ or $a=3$. This implies that $n=3$ or $n=5$.

Thus, across all cases, we must have $n=3$ or $n=5$, which we shall now solve separately. When $n=3$,
\[(m-6)(m-9)=m^2 - 15m+54=0\]to which the only possible solutions are $m=6$ and $m=9$. When $n=5$,
\[(m-9)(m-54)=m^2-63m+486 = 0\]to which the only solutions are $m=9$ and $m=54$. Thus, all solutions to the original equation must take the claimed forms, and we are done.
Z K Y
N Quick Reply
G
H
=
a